Wie weit können wir zwei gebundene Quarks trennen?

Wenn wir versuchen, zwei zu einem Meson oder einem Hadron verbundene Quarks zu trennen, wird die Energie im Gluonfeld schließlich groß genug sein, um ein Quark-Antiquark-Paar hervorzubringen.

Wie weit können wir dieses Gluonenfeld dehnen, bevor es „einrastet“? Was ist die durchschnittliche Entfernung? Ich kann es anscheinend nirgends finden.

Etwa ein Fermi, siehe hier: arxiv.org/abs/hep-lat/0505012 (zB Abb.22)

Antworten (2)

Aus Gitterrechnungen (siehe String Tension of Quark-Anti-Quark Pairs in Lattice QCD ) hat man herausgefunden, dass die String Tension der Quarks, im Fall von Pionen, gegeben ist durch

σ 460   M e v
was einer Länge von entspricht 2.7   f e r m ich .

Im Falle des Charmoniums ( c ¯ c ), die Spannung (siehe Charmonium-Potential aus Vollgitter-QCD ) ist nahe bei

σ 394   M e v
das heißt, eine Länge von 3.1   f e r m ich .

Für einen Bottomonium-Zustand ( b ¯ b ), ist die beste Schätzung, die ich gefunden habe (siehe Bottomonium-Zustände im Vergleich zu jüngsten experimentellen Beobachtungen im QCD-inspirierten Potenzialmodell ) .

σ 410   M e v
dh, 3   f e r m ich .

Diese Zahl ist zu groß, siehe meine Antwort hier: physical.stackexchange.com/questions/238218/…
@Thomas Woher hast du die 1   G e v / f m Wert? Sie können in meinen Links sehen (die neuer sind als Ihre), dass die LQCD-Ergebnisse näher beieinander liegen 400   M e v / f m ...
(460 MeV)^2=1 GeV/fm
@Thomas Welp, laut Wolfram Alpha ist es eher so 2.7   f m .
(460 MeV)^2 = (460 MeV)/(200 MeV*fm) = 460 MeV * 2,3/fm = 1 GeV/fm
@Thomas, da ist eine seltsame Algebra im Gange: P (vielleicht fehlt ein Quadrat?) Ich sehe sowieso nicht, wie irgendetwas davon für die Frage oder meine Antwort relevant ist (und auf jeden Fall ein mögliches Ergebnis dafür Diese Art von Fragen ist eine Schätzung, daher ist es vollkommen vernünftig, dass Sie und ich zwei unterschiedliche Ergebnisse erhalten, meine doppelt so groß wie Ihre)
1) Da die Saitenbruchskala in GeV bekannt ist, müssen wir die Saitenspannung von Energie im Quadrat in Energie/Entfernung umrechnen. Kein Geheimnis hier. 2) Es gibt einen seltsamen Fehler in woflram alpha. 460 MeV=2,3/fm nicht 2,7/fm, aber das ist ja auch keine große Sache. 3) Die Saitenbruchskala ist nicht um den Faktor drei unsicher, sie ist um (10-15)% unsicher, siehe den von mir zitierten Arxive-Preprint.
@ Thomas 1) du hast geschrieben (460 MeV) ^ 2 = (460 MeV)/(200 MeV fm), wenn Sie wahrscheinlich (460 MeV) ^ 2 = (460 MeV) meinten 2 /(200 MeV fm) (was auch immer, nicht wichtig) 2) ja, komisch, oder?. 3) eine "Skala" ist immer um mehr als a unsicher 10 fünfzehn % , weil es schließlich eine Waage ist . Auch hier stimmt Ihr Artikel nicht mit meinem überein, daher ist es für uns sinnlos, darauf zu verweisen: Auch ich kann sagen "siehe den Vorabdruck, den ich zitiere", und wäre nicht richtiger / falscher als Sie ...
1) Ja. 3) Sie zitieren ein Papier für die Saitenspannung (natürlich richtig); Ich zeige, wie man dies in einen Saitenbruchabstand umwandelt (andere Zahl, aber gut bestimmt). Das von mir zitierte Papier bestätigt, dass dies die richtige Zahl ist, weil sie SOWOHL die Saitenspannung als auch den Saitenbruchabstand berechnen.
@Thomas Oh, jetzt verstehe ich, was du meinst! sorry dafür, ich habe dich nur falsch verstanden. Es dachte, Sie sagten, dass die Saitenspannung um den Faktor zwei abweicht, nicht der Bremsweg. Ich habe nur die Spannung in Entfernung umgerechnet und dabei die Masse/Energie-Details vernachlässigt, und deshalb habe ich gesagt, dass die Zahl eine Schätzung ist. Aber Sie haben Recht, dass Ihre Berechnung genauer ist (obwohl ich immer noch denke, dass jede mögliche Antwort auf diese Frage mindestens um den Faktor zwei unsicher ist, sodass die Details in gewissem Maße vernachlässigt werden können : Wir sprechen hier von QM , also ist die "Entfernung" ungewiss )
Der QCD-String hat schwere Quarks an seinen Endpunkten, sodass die Stringlänge bis auf eine Unsicherheit definiert ist 1 / m b 0,05 fm.

Von der HyperPhysics-Site zu Quarks :

Es wird postuliert, dass sie tatsächlich mit der Entfernung um etwa 1 GeV pro Fermi zunehmen kann . Ein freies Quark wird nicht beobachtet, da die Energie zu dem Zeitpunkt, zu dem die Trennung auf einer beobachtbaren Skala liegt, weit über der Paarerzeugungsenergie für Quark-Antiquark-Paare liegt. Für die U- und D-Quarks betragen die Massen 10 MeV, so dass eine Paarbildung für Entfernungen auftreten würde, die viel kleiner als ein Fermi sind . In Kollisionsexperimenten mit sehr hoher Energie würde man viele Mesonen (Quark-Antiquark-Paare) erwarten, und genau das wird beobachtet.

Soweit ich das beurteilen kann, ist die tatsächliche Energiezunahme pro Fermi nicht mit hoher Genauigkeit bekannt, da die Reichweite so unglaublich gering ist. Wenn man bedenkt, dass die starke Kernkraft nur in Entfernungen von weniger als an Wechselwirkungen teilnehmen kann 10 fünfzehn m ist verständlich, dass es keine genauen Zahlen für den maximalen Abstand gibt, den zwei Quarks haben können.

Prüfen. Aber Sie lassen Hadronen aus dem Vakuum springen, also, sagen wir, ein π⁰, also dann sehr grob 0,135 Fermis ... mehr als ein Zehntel Fermi, wenn es sein muss.